Show that the functional $F$ on $L_p(-1,1)$ given by $ F(f) = int_{-1}^0 f(t)dt - int_{0}^1 f(t)dt $ is...












1












$begingroup$


I have to show that the functional $F$: $$ F(f) = int_{-1}^0 f(t)dt - int_{0}^1 f(t)dt $$



on the space $L_p(-1, 1), pgeqslant 1 $, is continuous.



Solution:



I need to get an inequality $$ vert F(f) vert le M left(int_{-1}^1 bigvert f(t)big vert ^p dtright)^{1/p}.$$



Here $M$ is a constant and $M>0$.



I started:



$$bigvert F(f) bigvert=leftvert int_{-1}^0 f(t)dt - int_{0}^1 f(t)dt rightvert .$$



And here I stopped. Next I have to evaluate these two integrals by new $int_{-1}^1 g(t)dt$ function and here I have a problem with this. The next step is to use the Hölder's inequality to get the inequality I need to show (with $M>0$).



Question:



How to find $int_{-1}^1 g(t)dt$?










share|cite|improve this question











$endgroup$








  • 2




    $begingroup$
    Is there a reason to write $$F(f)=int_{-1}^0,f(t),text{d}t+int_0^1,f(t),text{d}t$$ instead of just $$F(f)=int_{-1}^1,f(t),text{d}t,?$$ (That is, did you make any typo?)
    $endgroup$
    – Batominovski
    Dec 11 '18 at 20:49










  • $begingroup$
    Oh.. sorry, $-$ have to be there
    $endgroup$
    – Philip
    Dec 11 '18 at 20:52
















1












$begingroup$


I have to show that the functional $F$: $$ F(f) = int_{-1}^0 f(t)dt - int_{0}^1 f(t)dt $$



on the space $L_p(-1, 1), pgeqslant 1 $, is continuous.



Solution:



I need to get an inequality $$ vert F(f) vert le M left(int_{-1}^1 bigvert f(t)big vert ^p dtright)^{1/p}.$$



Here $M$ is a constant and $M>0$.



I started:



$$bigvert F(f) bigvert=leftvert int_{-1}^0 f(t)dt - int_{0}^1 f(t)dt rightvert .$$



And here I stopped. Next I have to evaluate these two integrals by new $int_{-1}^1 g(t)dt$ function and here I have a problem with this. The next step is to use the Hölder's inequality to get the inequality I need to show (with $M>0$).



Question:



How to find $int_{-1}^1 g(t)dt$?










share|cite|improve this question











$endgroup$








  • 2




    $begingroup$
    Is there a reason to write $$F(f)=int_{-1}^0,f(t),text{d}t+int_0^1,f(t),text{d}t$$ instead of just $$F(f)=int_{-1}^1,f(t),text{d}t,?$$ (That is, did you make any typo?)
    $endgroup$
    – Batominovski
    Dec 11 '18 at 20:49










  • $begingroup$
    Oh.. sorry, $-$ have to be there
    $endgroup$
    – Philip
    Dec 11 '18 at 20:52














1












1








1


1



$begingroup$


I have to show that the functional $F$: $$ F(f) = int_{-1}^0 f(t)dt - int_{0}^1 f(t)dt $$



on the space $L_p(-1, 1), pgeqslant 1 $, is continuous.



Solution:



I need to get an inequality $$ vert F(f) vert le M left(int_{-1}^1 bigvert f(t)big vert ^p dtright)^{1/p}.$$



Here $M$ is a constant and $M>0$.



I started:



$$bigvert F(f) bigvert=leftvert int_{-1}^0 f(t)dt - int_{0}^1 f(t)dt rightvert .$$



And here I stopped. Next I have to evaluate these two integrals by new $int_{-1}^1 g(t)dt$ function and here I have a problem with this. The next step is to use the Hölder's inequality to get the inequality I need to show (with $M>0$).



Question:



How to find $int_{-1}^1 g(t)dt$?










share|cite|improve this question











$endgroup$




I have to show that the functional $F$: $$ F(f) = int_{-1}^0 f(t)dt - int_{0}^1 f(t)dt $$



on the space $L_p(-1, 1), pgeqslant 1 $, is continuous.



Solution:



I need to get an inequality $$ vert F(f) vert le M left(int_{-1}^1 bigvert f(t)big vert ^p dtright)^{1/p}.$$



Here $M$ is a constant and $M>0$.



I started:



$$bigvert F(f) bigvert=leftvert int_{-1}^0 f(t)dt - int_{0}^1 f(t)dt rightvert .$$



And here I stopped. Next I have to evaluate these two integrals by new $int_{-1}^1 g(t)dt$ function and here I have a problem with this. The next step is to use the Hölder's inequality to get the inequality I need to show (with $M>0$).



Question:



How to find $int_{-1}^1 g(t)dt$?







functional-analysis functions inequality continuity lp-spaces






share|cite|improve this question















share|cite|improve this question













share|cite|improve this question




share|cite|improve this question








edited Dec 11 '18 at 21:27









Batominovski

33.1k33293




33.1k33293










asked Dec 11 '18 at 20:44









PhilipPhilip

857




857








  • 2




    $begingroup$
    Is there a reason to write $$F(f)=int_{-1}^0,f(t),text{d}t+int_0^1,f(t),text{d}t$$ instead of just $$F(f)=int_{-1}^1,f(t),text{d}t,?$$ (That is, did you make any typo?)
    $endgroup$
    – Batominovski
    Dec 11 '18 at 20:49










  • $begingroup$
    Oh.. sorry, $-$ have to be there
    $endgroup$
    – Philip
    Dec 11 '18 at 20:52














  • 2




    $begingroup$
    Is there a reason to write $$F(f)=int_{-1}^0,f(t),text{d}t+int_0^1,f(t),text{d}t$$ instead of just $$F(f)=int_{-1}^1,f(t),text{d}t,?$$ (That is, did you make any typo?)
    $endgroup$
    – Batominovski
    Dec 11 '18 at 20:49










  • $begingroup$
    Oh.. sorry, $-$ have to be there
    $endgroup$
    – Philip
    Dec 11 '18 at 20:52








2




2




$begingroup$
Is there a reason to write $$F(f)=int_{-1}^0,f(t),text{d}t+int_0^1,f(t),text{d}t$$ instead of just $$F(f)=int_{-1}^1,f(t),text{d}t,?$$ (That is, did you make any typo?)
$endgroup$
– Batominovski
Dec 11 '18 at 20:49




$begingroup$
Is there a reason to write $$F(f)=int_{-1}^0,f(t),text{d}t+int_0^1,f(t),text{d}t$$ instead of just $$F(f)=int_{-1}^1,f(t),text{d}t,?$$ (That is, did you make any typo?)
$endgroup$
– Batominovski
Dec 11 '18 at 20:49












$begingroup$
Oh.. sorry, $-$ have to be there
$endgroup$
– Philip
Dec 11 '18 at 20:52




$begingroup$
Oh.. sorry, $-$ have to be there
$endgroup$
– Philip
Dec 11 '18 at 20:52










1 Answer
1






active

oldest

votes


















2












$begingroup$

Using the Triangle Inequality, we have
$$big|F(f)big|leq int_{-1}^{+1},big|f(x)big|,text{d}xtext{ for each }fin L^pbig((-1,+1)big),.$$
By Hölder's Inequality,
$$begin{align}int_{-1}^{+1},big|f(x)big|,text{d}x&=int_{-1}^{+1},big|f(x)big|cdot 1,text{d}x\&leq left(int_{-1}^{+1},big|f(x)big|^p,text{d}xright)^{frac{1}{p}},left(int_{-1}^{+1},1^q,text{d}xright)^{frac{1}{q}}text{ for each }fin L^pbig((-1,+1)big),,end{align}$$
where $qin[1,infty]$ is such that $dfrac1p+dfrac1q=1$. This shows that
$$big|F(f)big|leq |f|_p,2^{frac{1}{q}}=2^{1-frac1p},|f|_ptext{ for each }fin L^pbig((-1,+1)big),,$$
or
$$|F|_{L^p_text{op}}leq 2^{1-frac1p},.$$
By taking $f:(-1,+1)tomathbb{C}$ to be the step function
$$f(x)=begin{cases}-1&text{if }-1<x<0,,\+1&text{if }0leq x <+1,,end{cases}$$
we can see that the operator norm $|F|_{L^p_text{op}}$ of $F$ does indeed equal $2^{1-frac1p}$ (thus, $F$ is a bounded linear functional, whence continuous). With a little tweak, this result is also true for $p=infty$, not just $pin[1,infty)$.






share|cite|improve this answer











$endgroup$













    Your Answer





    StackExchange.ifUsing("editor", function () {
    return StackExchange.using("mathjaxEditing", function () {
    StackExchange.MarkdownEditor.creationCallbacks.add(function (editor, postfix) {
    StackExchange.mathjaxEditing.prepareWmdForMathJax(editor, postfix, [["$", "$"], ["\\(","\\)"]]);
    });
    });
    }, "mathjax-editing");

    StackExchange.ready(function() {
    var channelOptions = {
    tags: "".split(" "),
    id: "69"
    };
    initTagRenderer("".split(" "), "".split(" "), channelOptions);

    StackExchange.using("externalEditor", function() {
    // Have to fire editor after snippets, if snippets enabled
    if (StackExchange.settings.snippets.snippetsEnabled) {
    StackExchange.using("snippets", function() {
    createEditor();
    });
    }
    else {
    createEditor();
    }
    });

    function createEditor() {
    StackExchange.prepareEditor({
    heartbeatType: 'answer',
    autoActivateHeartbeat: false,
    convertImagesToLinks: true,
    noModals: true,
    showLowRepImageUploadWarning: true,
    reputationToPostImages: 10,
    bindNavPrevention: true,
    postfix: "",
    imageUploader: {
    brandingHtml: "Powered by u003ca class="icon-imgur-white" href="https://imgur.com/"u003eu003c/au003e",
    contentPolicyHtml: "User contributions licensed under u003ca href="https://creativecommons.org/licenses/by-sa/3.0/"u003ecc by-sa 3.0 with attribution requiredu003c/au003e u003ca href="https://stackoverflow.com/legal/content-policy"u003e(content policy)u003c/au003e",
    allowUrls: true
    },
    noCode: true, onDemand: true,
    discardSelector: ".discard-answer"
    ,immediatelyShowMarkdownHelp:true
    });


    }
    });














    draft saved

    draft discarded


















    StackExchange.ready(
    function () {
    StackExchange.openid.initPostLogin('.new-post-login', 'https%3a%2f%2fmath.stackexchange.com%2fquestions%2f3035792%2fshow-that-the-functional-f-on-l-p-1-1-given-by-ff-int-10-ftdt%23new-answer', 'question_page');
    }
    );

    Post as a guest















    Required, but never shown

























    1 Answer
    1






    active

    oldest

    votes








    1 Answer
    1






    active

    oldest

    votes









    active

    oldest

    votes






    active

    oldest

    votes









    2












    $begingroup$

    Using the Triangle Inequality, we have
    $$big|F(f)big|leq int_{-1}^{+1},big|f(x)big|,text{d}xtext{ for each }fin L^pbig((-1,+1)big),.$$
    By Hölder's Inequality,
    $$begin{align}int_{-1}^{+1},big|f(x)big|,text{d}x&=int_{-1}^{+1},big|f(x)big|cdot 1,text{d}x\&leq left(int_{-1}^{+1},big|f(x)big|^p,text{d}xright)^{frac{1}{p}},left(int_{-1}^{+1},1^q,text{d}xright)^{frac{1}{q}}text{ for each }fin L^pbig((-1,+1)big),,end{align}$$
    where $qin[1,infty]$ is such that $dfrac1p+dfrac1q=1$. This shows that
    $$big|F(f)big|leq |f|_p,2^{frac{1}{q}}=2^{1-frac1p},|f|_ptext{ for each }fin L^pbig((-1,+1)big),,$$
    or
    $$|F|_{L^p_text{op}}leq 2^{1-frac1p},.$$
    By taking $f:(-1,+1)tomathbb{C}$ to be the step function
    $$f(x)=begin{cases}-1&text{if }-1<x<0,,\+1&text{if }0leq x <+1,,end{cases}$$
    we can see that the operator norm $|F|_{L^p_text{op}}$ of $F$ does indeed equal $2^{1-frac1p}$ (thus, $F$ is a bounded linear functional, whence continuous). With a little tweak, this result is also true for $p=infty$, not just $pin[1,infty)$.






    share|cite|improve this answer











    $endgroup$


















      2












      $begingroup$

      Using the Triangle Inequality, we have
      $$big|F(f)big|leq int_{-1}^{+1},big|f(x)big|,text{d}xtext{ for each }fin L^pbig((-1,+1)big),.$$
      By Hölder's Inequality,
      $$begin{align}int_{-1}^{+1},big|f(x)big|,text{d}x&=int_{-1}^{+1},big|f(x)big|cdot 1,text{d}x\&leq left(int_{-1}^{+1},big|f(x)big|^p,text{d}xright)^{frac{1}{p}},left(int_{-1}^{+1},1^q,text{d}xright)^{frac{1}{q}}text{ for each }fin L^pbig((-1,+1)big),,end{align}$$
      where $qin[1,infty]$ is such that $dfrac1p+dfrac1q=1$. This shows that
      $$big|F(f)big|leq |f|_p,2^{frac{1}{q}}=2^{1-frac1p},|f|_ptext{ for each }fin L^pbig((-1,+1)big),,$$
      or
      $$|F|_{L^p_text{op}}leq 2^{1-frac1p},.$$
      By taking $f:(-1,+1)tomathbb{C}$ to be the step function
      $$f(x)=begin{cases}-1&text{if }-1<x<0,,\+1&text{if }0leq x <+1,,end{cases}$$
      we can see that the operator norm $|F|_{L^p_text{op}}$ of $F$ does indeed equal $2^{1-frac1p}$ (thus, $F$ is a bounded linear functional, whence continuous). With a little tweak, this result is also true for $p=infty$, not just $pin[1,infty)$.






      share|cite|improve this answer











      $endgroup$
















        2












        2








        2





        $begingroup$

        Using the Triangle Inequality, we have
        $$big|F(f)big|leq int_{-1}^{+1},big|f(x)big|,text{d}xtext{ for each }fin L^pbig((-1,+1)big),.$$
        By Hölder's Inequality,
        $$begin{align}int_{-1}^{+1},big|f(x)big|,text{d}x&=int_{-1}^{+1},big|f(x)big|cdot 1,text{d}x\&leq left(int_{-1}^{+1},big|f(x)big|^p,text{d}xright)^{frac{1}{p}},left(int_{-1}^{+1},1^q,text{d}xright)^{frac{1}{q}}text{ for each }fin L^pbig((-1,+1)big),,end{align}$$
        where $qin[1,infty]$ is such that $dfrac1p+dfrac1q=1$. This shows that
        $$big|F(f)big|leq |f|_p,2^{frac{1}{q}}=2^{1-frac1p},|f|_ptext{ for each }fin L^pbig((-1,+1)big),,$$
        or
        $$|F|_{L^p_text{op}}leq 2^{1-frac1p},.$$
        By taking $f:(-1,+1)tomathbb{C}$ to be the step function
        $$f(x)=begin{cases}-1&text{if }-1<x<0,,\+1&text{if }0leq x <+1,,end{cases}$$
        we can see that the operator norm $|F|_{L^p_text{op}}$ of $F$ does indeed equal $2^{1-frac1p}$ (thus, $F$ is a bounded linear functional, whence continuous). With a little tweak, this result is also true for $p=infty$, not just $pin[1,infty)$.






        share|cite|improve this answer











        $endgroup$



        Using the Triangle Inequality, we have
        $$big|F(f)big|leq int_{-1}^{+1},big|f(x)big|,text{d}xtext{ for each }fin L^pbig((-1,+1)big),.$$
        By Hölder's Inequality,
        $$begin{align}int_{-1}^{+1},big|f(x)big|,text{d}x&=int_{-1}^{+1},big|f(x)big|cdot 1,text{d}x\&leq left(int_{-1}^{+1},big|f(x)big|^p,text{d}xright)^{frac{1}{p}},left(int_{-1}^{+1},1^q,text{d}xright)^{frac{1}{q}}text{ for each }fin L^pbig((-1,+1)big),,end{align}$$
        where $qin[1,infty]$ is such that $dfrac1p+dfrac1q=1$. This shows that
        $$big|F(f)big|leq |f|_p,2^{frac{1}{q}}=2^{1-frac1p},|f|_ptext{ for each }fin L^pbig((-1,+1)big),,$$
        or
        $$|F|_{L^p_text{op}}leq 2^{1-frac1p},.$$
        By taking $f:(-1,+1)tomathbb{C}$ to be the step function
        $$f(x)=begin{cases}-1&text{if }-1<x<0,,\+1&text{if }0leq x <+1,,end{cases}$$
        we can see that the operator norm $|F|_{L^p_text{op}}$ of $F$ does indeed equal $2^{1-frac1p}$ (thus, $F$ is a bounded linear functional, whence continuous). With a little tweak, this result is also true for $p=infty$, not just $pin[1,infty)$.







        share|cite|improve this answer














        share|cite|improve this answer



        share|cite|improve this answer








        edited Dec 12 '18 at 0:55

























        answered Dec 11 '18 at 21:14









        BatominovskiBatominovski

        33.1k33293




        33.1k33293






























            draft saved

            draft discarded




















































            Thanks for contributing an answer to Mathematics Stack Exchange!


            • Please be sure to answer the question. Provide details and share your research!

            But avoid



            • Asking for help, clarification, or responding to other answers.

            • Making statements based on opinion; back them up with references or personal experience.


            Use MathJax to format equations. MathJax reference.


            To learn more, see our tips on writing great answers.




            draft saved


            draft discarded














            StackExchange.ready(
            function () {
            StackExchange.openid.initPostLogin('.new-post-login', 'https%3a%2f%2fmath.stackexchange.com%2fquestions%2f3035792%2fshow-that-the-functional-f-on-l-p-1-1-given-by-ff-int-10-ftdt%23new-answer', 'question_page');
            }
            );

            Post as a guest















            Required, but never shown





















































            Required, but never shown














            Required, but never shown












            Required, but never shown







            Required, but never shown

































            Required, but never shown














            Required, but never shown












            Required, but never shown







            Required, but never shown







            Popular posts from this blog

            Plaza Victoria

            In PowerPoint, is there a keyboard shortcut for bulleted / numbered list?

            How to put 3 figures in Latex with 2 figures side by side and 1 below these side by side images but in...